Calculus (3rd Edition)

Published by W. H. Freeman
ISBN 10: 1464125260
ISBN 13: 978-1-46412-526-3

Chapter 3 - Differentiation - 3.7 The Chain Rule - Exercises - Page 146: 79

Answer

(a) $14.4\pi$ (b) $2.304\pi$

Work Step by Step

Given $ r= 0.4t $, since $V= \dfrac{4}{3}\pi r^3$, then \begin{align*} \frac{d V}{d t}&=\frac{d V}{d r} \frac{d r}{d t}\\ &=( 4 \pi r^{2}) \cdot(0.4)\\ &=1.6 \pi r^{2} \end{align*} (a) For $r= 3$, we get \begin{align*} \frac{d V}{d t}\bigg|_{r=3} &=1.6 \pi (3)^{2}\\ &= 14.4\pi \end{align*} (b) For $t= 3$, we have $r= 1.2$ \begin{align*} \frac{d V}{d t}\bigg|_{t=3} &=1.6 \pi (1.2)^{2}\\ &= 2.304\pi \end{align*}
Update this answer!

You can help us out by revising, improving and updating this answer.

Update this answer

After you claim an answer you’ll have 24 hours to send in a draft. An editor will review the submission and either publish your submission or provide feedback.